You planted 60 seeds seeds.80% actually grow. how many seeds grew? To word it another way, what is the amount of seeds that grew if 80% of 60% seeds grew?

Answers

Answer 1

Answer:

48

Step-by-step explanation:

0.8 * 60 = 48


Related Questions

What are rational numbers but not natural numbers?; Which set of numbers are rational numbers but not integers whole numbers or natural numbers?; Does the set of rational numbers contain the set of natural numbers?; What is a rational number that is not a real number?

Answers

The distinction between them is that an integer is a negative number without a decimal point.

What is integers?

All whole numbers and negative numbers are considered integers. This means if we include negative numbers together with whole numbers, we form a set of integers.

Positive, negative, and zero are all examples of integers.

The Latin word "integer" signifies "whole" or "intact."

As a result, fractions and decimals are not included in integers. In this article, let's learn more about integers, their definition, and their characteristics.

According to our question-

The group of integers that are neither whole nor a natural number are:

As previously noted, the third set of integers lacks a decimal point;

However, the difference between them is that; a negative number without a decimal point is referred to as an integer

From the provided set: only fit into the description of an integer, because

It does not have a decimal point

And that is unfavorable.

learn more about integers click here:

https://brainly.com/question/929808

#SPJ4

PLEASEEE HELP THIS IS 3 GRAFE LEVELS ABOVE ME!
Check your answer to problem 4 by solving it using a different strategy Show your work.​

Answers

Answer:

you are right

n=1

Step-by-step explanation:

1/2(-3/2n+1)=3/4-n

-3/4n+1/2=3/4-n

-3/4n+2/4=3/4-4/4n

+3/4n               +3/4n

2/4=3/4-1/4n

-3/4. -3/4

-1/4= -1/4n

/-1/4.  /-1/4

1=n

to check i normally just pop in the value (i know no other way in all my years of school)

1/2(-3/2(1)+1)=3/4-1

1/2(-3/2+1=3/4-1

1/2(-1/2)=-1/4

-1/4= -1/4

hopes this helps please mark brainliest

Which of the following are solutions to the inequality below? Select all that apply.
4>q/23
q = 138
q = 46
q = 69
q = 92

Answers

To get q by itself, we multiply both sides by 23
We get
92>q
q can be 46 or 69 because they are both less than 92

A quadrilateral is shown in the diagram.
Which expression represents the perimeter of the figure in the diagram?
(A) 10 a+2
(B) 20 a-2
(c) 16 a+12
(D) 15 a-7

Answers

Step-by-step explanation:

F males + weua yvy did / How does Landman’s new eclectic phase differ from Blondel’s behaviouralist phase?

a popular brand of pen is available in 5 colors and 4 writing tips. how many different choices of pens do you have with this brand?

Answers

If a popular brand of pen is available in 5 colors and 4 writing tips. The  number of different choices of pens do you have with this brand is: 20 choices.

How to find the number of different choices of pen?

Using this formula to find the number of different choices of pen

Number of different choices of pen = Number of  color × Number of writing tips

Where:

Number of color = 5

Number of writing tips = 4

Let plug in the formula

Number of different choices of pen = 5 × 4

Number of different choices of pen = 20 choices

Therefore the number of different choices of pen  is 20 choices.

Learn more about number of different choices of pen here:https://brainly.com/question/16075452

#SPJ1

5-55.
Additional Challenge: At the beginning of 1990, oil prices were $20 a barrel. Some oil investors predicted that the price
of oil would increase by $2.25 a barrel per year.
In the beginning of 2005, the price of oil was $30 a barrel. With increasing demand for oll around the world, oil investors in
2005 predicted that the price of oil would increase by $5.00 a barrel each year.
DILOIL
a. Let a represent the number of years since 2005. Write an equation that predicts the price of oil, y, using the information available in 2005.
b. Investors in 1990 did not have the benefit of the 2005 information. Write an equation that represents the prediction made in 1990, using the same variables as in
part (a). Remember that a represents the number of years since 2005.
c. Use the equations you wrote in parts (a) and (b) to determine when the cost of a barrel of oil would be the same for both price predictions.
d. In the spring of 2011, a barrel of oil was selling for about $112. Which prediction was closer? Was it a pretty good prediction?

Answers

a) The linear function for the 2005 estimate is given as follows: y = 30 + 5x.

b) The linear function for the 1990 estimate is given as follows: y = 53.75 + 2.25x.

c) They would be predicted to have the same cost in the year of 2013.

d) The 1990 prediction was closer, however it still had a large residual, hence neither was good enough.

How to define the linear functions?

The linear functions are defined in the slope-intercept format, given as follows:

y = mx + b.

In which:

m is the slope, representing the rate of change for the price.b is the y-intercept, representing the initial cost in the reference year.

Taking 2005 as the estimate, the function is given as follows:

y = 30 + 5x.

Taking 1990 as the estimate, the equation would be of:

y = 20 + 2.25x.

The estimate for 2005 would be of:

y = 20 + 2.25(15) = 53.75.

Hence the equation taking 2005 as the estimate would be of:

y = 53.75 + 2.25x.

The costs would be the same when:

30 + 5x = 53.75 + 2.25x.

2.75x = 23.75

x = 23.75/2.75

x = 8.63 -> + 2005 -> year of 2013.

The estimates for 2011(6 years after 2005) are given as follows:

y = 30 + 5x = 30 + 5(6) = $60.y = 53.75 + 2.25(6) = $67.25. (better estimate but not good enough).

More can be learned about linear functions at https://brainly.com/question/24808124

#SPJ1

For the functions f(t) = 5t and g(t) = sin(t) defined on 0≤t<[infinity], compute f * g in two different ways.a. By directly evaluating the integral in the definition of f * g.b. By computing L−1{F(s)G(s)} where F(s)=L{f(t)} and G(s)=L{g(t)}.

Answers

The laplace inverse transformation L−1{F(s)G(s)} =  5 [tex]\frac{2s}{s^{2} + 1^{2} }[/tex]  

If L{f(t)} = F(s), then the inverse Laplace transform of F(s) is

L−1{F(s)} = f(t). (1)

The inverse transform L−1is a linear operator:

L−1{F(s) + G(s)} = L−1{F(s)} + L−1{G(s)}, (2)

andL−1{cF(s)} = cL−1{F(s)}, (3)

for any constant c

given functions are

f(t) =5t ,g(t) = sin(t)

L(f(t)) = F(s)

L{F(s)*G(s)} = L^-1{5tsin(t)}

   =5L^-1{tsin(t)}

   =5L^-1{[tex]\frac{d}{ds}[/tex] tsin(t)}

    = 5 [tex]\frac{2s}{s^{2} + 1^{2} }[/tex]  

To learn more about inverse Laplace transform :

https://brainly.com/question/1675085

#SPJ4

$6,000 for six years at 8½% compounded daily will grow to:
Multiple Choice
$9,060.00
$9,788.81
$9,991.15

Answers

It will grow to $9991.15

A car that travels 20 miles in 1/2 hour at constant speed is traveling at the same speed as a car that travels 30 miles. In how many hours (at a constant speed)

Answers

The required time to travel 30 miles is given as 45 minutes and 3/4 hours.

Given that,
A car that travels 20 miles in 1/2 hour at constant speed is traveling at the same speed as a car that travels 30 miles.

What is simplification?

The process in mathematics to operate and interpret the function to make the function or expression simple or more understandable is called simplifying and the process is called simplification.

here,
Speed = Distance/time
Speed = 20 / [1/2]
Speed = 40 miles per hour,
Time = Distance / Speed
Time = 30 / 40
Time = 3 / 4 hour

Thus, the required time to travel 30 miles is given as 45 minutes and 3/4 hours.

Learn more about simplification here:

https://brainly.com/question/12501526

#SPJ1

What is ? x 5 = 9 for 5th grade

Answers

Answer:

The answer is 1.8

Step-by-step explanation:

Let the number be y

So,

y × 5 = 9

5y = 9

Divide both side by 5

5y/5 = 9/5

y = 9/5 = 1.8

Thus, The value of y is 1.8

Order the answers of these following expressions from least to greatest. -13 + 12= . -3 - (-2) + 3= . -3 + 9= . 9 + (-15)= . 13 - (-13)= .

Answers

The answers of the given expressions ordered from least to greatest is

-6, -1, 2, 6, 26

Ordering the answers of expressions from least to greatest

From the question, we are to order the answers of the given expressions from the least to greatest

To do this, we will evaluate the expressions

Evaluating -13 + 12 =

-13 + 12 = -1

Evaluating -3 - (-2) + 3

-3 - (-2) + 3

-3 + 2 + 3 = 2

Evaluating -3 + 9 =

-3 + 9 = 6

Evaluating 9 + (-15)

9 + (-15)

9 - 15 = -6

Evaluating 13 - (-13)

13 - (-13)

13 + 13 = 26

Hence, ordering the answers from least to greatest, we get

-6, -1, 2, 6, 26

Learn more on Ordering the answers of expressions from least to greatest here: https://brainly.com/question/22559078

#SPJ1

hese are curves obtained by the intersection of right circular cone and a plane.
a. Parabola
b. Ellipse
c. Hyperbola
d. Conic sections

Answers

Answer: conic sections

Step-by-step explanation: the other options are a part of conic sections that are named depending on the angle of the plane relative to the cone, and the intersection.

The domestic price of shoes is $80. After trade the price of a pair of shoes is $60. After trade this country will import

Answers

The trade this country will import is given by the option (C) 300 pair of shoes.

The number of shoes exported will be calculated by the formula -

Import = quantity demanded - quantity supplied. The formula is based on the concept that the import of shoes will be carried out once the manufacturing is more than the demand.

The graph indicates the quantity demanded is 1000 and the quantity supplied is 1300. Keep the values in formula to find imports.

Imports = 1300 - 1000

Performing subtraction on Right Hand Side of the equation

Imports = 300

Hence, the import is 300.

Learn more about import -

https://brainly.com/question/24473707

#SPJ4

The complete question is attached in figure.

How do you teach regrouping to 2nd graders?

Answers

Answer:Pass out the pennies and the dimes to your students. Write a couple of problems on the board that will use regrouping. As you write them, write “ones” over the ones column. Write “tens” over the tens column. Have students take out their coins for each problem.

Step-by-step explanation:

an amusement park sold 374 discount tickets and 306 full price tickets . What percentage of the tickets sold were discount tickets

Answers

Answer:55%

Step-by-step explanation:So, first find the total number of tickets sold 374+306=680. Now, divide the number of discount tickets by the total number and multiply the result by 100% to express the result as a percentage. Percentage of discount tickets=(374/680)100%=55%. So, 55% percent of the tickets sold at the amusement park were discounted.

At the time of her​ grandson's birth, a grandmother deposits 14000 in an account that pays 9.5% compounded monthly. What will be the value of the account at the​ child's twenty-first​ birthday, assuming that no other deposits or withdrawals are made during this​ period?

Answers

The value of the account at the​ child's twenty-first​ birthday is 102125.44

How to determine the value of the account at the​ child's twenty-first​ birthday?

We can use the compound interest formula to determine the value of the account (A) on the​ child's twenty-first​ birthday:

A = P (1 + r/n)^(nt)

Where:

P is the initial amount of money deposited in the account

r is the interest rate

n is the number of times the interest is compounded per year

t is the number of years

Given: P = 14000, r = 9.5% = 0.095, t = 21, n = 12 (i.e. 12 months per year)

Substitututing into A = P (1 + r/n)^(nt):

A = 14000 (1 +  0.095/12)^(12×21)

A = 14000 (1 +  0.095/12)^(252)

A = 102125.44

Learn more about compound interest on:

https://brainly.com/question/24274034

#SPJ1

Find the slope of the line that passes through (5, 9) and (3, -5). Write your answer in simplest form.

Answers

Therefore the slope of the line passes through  (5, 9) and (3, -5) comes out to be 7.

What is slope?

By computing the ratio of the vertical to horizontal distance (rise over run) between two points, the slope—a numerical value that defines how steep a line is—is often calculated.

Here,

The slope of line that passes through  (5, 9) and (3, -5)
Slope= gradient between the two points

Slope= (y2-y1)/(x2-x1)

Where y2= -5

Y1= 9

X2= 3

X1= 5

=> (-5-9)/(3-5)

=> -14/-2

=> 7

Therefore the slope of the line passes through  (5, 9) and (3, -5) comes out to be 7.

To know more about slope , visit

https://brainly.com/question/3605446

#SPJ1

The diameter of a quarter is about 1 in.
You trace around the edge of the quarter on a sheet of paper.
What is the area of the circle on the paper?

Use 3.14 as an approximation for π. Round your answer to the nearest tenth.

Answers

Answer: The area is going to be 0.8

Step-by-step explanation:  The diameter is 1

You need the radius of the circle which is half the diameter. So the Radius(R) is 0.5.

After you find the radius of the circle to find the area of the circle you use the formula pie (R)^2. You are using 3.14

3.14(0.5)^2= 0.785

you then want to round your answer to the nearest tenth so the 8 turns the 7 into an 8.

So, the Area is 0.785 but after rounding it is 0.8.

Hope that helped :)

A motorist claims that the South Boro Police issue an average of 60 speeding tickets per day. These data show the number of speeding tickets issued each day for a randomly selected period of 30 days. Assume sigma = 13.42. Is there enough evidence to reject the motorists claim at alpha = 0.05 ? Use either the P-value Method or the Traditional Method. 72 45 36 68 69 71 57 60 83 26 60 72 58 87 48 59 60 56 64 68 42 57 57 58 63 49 73 75 42 63

Answers

According to a driver, the South Boro Police issue 60 speeding penalties on average each day. These numbers display the daily average number of speeding fines issued over a randomly chosen 30-day period. At alpha = 0.05, there is not enough data to disprove the motorists' assertion.

Traditional Method:

H0: μ = 60

H1: μ ≠ 60

n = 30

Sample mean = 58.9

Standard Deviation = 13.42

t-statistic = -1.85

Number of Degrees of Freedom = 30 - 1 = 29

α = 0.05

Critical t-value = ±2.045

Since -1.85 does not lie in the rejection region, we fail to reject the null hypothesis.

Therefore, there is not enough evidence to reject the motorists claim at alpha = 0.05.

Learn more about alpha here

https://brainly.com/question/28170151

#SPJ4

Which of the following is an identity?
A. csc²x + cot²x = 1
B. (cscx + cotx)2 = 1
C. sin²x-cos²x = 1
D. sin²x sec²x + 1 = tan²x csc²x

Answers

Answer:

Step-by-step explanation:

it is D as

L.H.S.=sin²x sec²x+1

[tex]\frac{sin^2x}{cos^2x} +1\\=\frac{sin^2x+cos^2x}{cos^2x} \\=\frac{1}{cos^2x} \\=sec^2x[/tex]

again

[tex]R.H.S.=tan^2x csc^2x\\=\frac{sin^2x}{cos^2x} \times \frac{1}{sin^2x} \\=\frac{1}{cos^2x} \\=sec^2x[/tex]

L.H.S.=R.H.S.

what is a proportional relationship between x and y; proportional relationship calculator; proportional relationship formula; what is a proportional relationship example; proportional relationships 7th grade; is y=x/2 a proportional relationship; proportional relationship example problems

Answers

The proportional equation has a general form. The proportional relation is:

                                              y = k/x

The letters y and x are variables that can vary, and the letter k is the constant of proportionality. A constant of proportionality is a constant ratio between the y and x variables. The constant of proportionality can be found by calculating y/x.

A proportional relationship is a relationship between two variables in which as one variable changes, increases, or decreases, the other variable changes at a constant rate. A constant ratio is considered a constant ratio between two variables. Proportional relationships can be identified by proportional equations, graphs, or tables. The proportionality equations discussed in this lesson have the general form: The graph is displayed as a straight line. The table is pairs of numbers with a constant ratio between the pairs.

A linear relationship is written as: y = mx + b, where m is the slope and b is the y-intercept.A direct proportional relation is written as y = k/x. where k is the constant of proportionality. A direct proportional relationship is a linear relationship where k is the slope and the y-intercept is zero.

Therefore, only linear relationships with zero y-intercept are directly proportional.

For example:

If 1 hat is $20, 2 hats are $40, 3 hats are $60, and so on. This is a direct share. If you triple the number of hats, the total amount will triple.

For 1 hut and $30 fee, 2 hut and $50 fee, 3 hut and $70 fee. This is a linear relationship, with each additional hat ordered increases the price by $20. However, due to a one-time processing fee of $10, the total price of 3 hats will not be 3 times the total price of 1 hat.

Learn more about proportional relationship:

https://brainly.com/question/29696241

#SPJ4

Can someone help me with this, please?

Answers

π√π÷√π  is an irrational number. So option (a) is required answer.

What is irrational number?

A real number that cannot be stated as a ratio of integers is said to be irrational; an example of this is the number 2. Any irrational number, such as p/q, where p and q are integers, q, cannot be expressed as a ratio. Once more, an irrational number's decimal expansion is neither ending nor recurrent.

Examples of irrational number: √3, ∛2, π

π is an irrational because it is neither ending nor recurrent.

Solving each part of options, we get

π√π÷√π = π

√25 = 5

√π÷π = 1

2√2 ÷ √2 = 2

As most of the option give rational number only one option gives irrational number which is π

We get π from option (a)

So, option (a) is correct option.

Learn more about irrational number here:

https://brainly.com/question/43641

#SPJ1

a baseball coach uses a pitching machine to simulate pop flies during practice. the quadratic function f(x)

Answers

The time taken by the baseball in the air if the ball is not caught is 4.513 seconds.

Here we have to find the time taken by the baseball.

Given data:

The quadratic function is given:

f(x) = 16x² + 70x + 10

To find the value of x we have to formula:

x = ±b [tex]\sqrt{b^{2} - 4ac}[/tex] / 2a

from the equation we get the value of a, b, and c as:

a = -16

b = 70

c = -10

now putting in the equation we have:

x = -70 ±[tex]\sqrt{70^{2} -4(-16)(10)}[/tex] / 2(-16)

  = -70 ±[tex]\sqrt{4900 + 640}[/tex] / -32

  = -70 ± √5540 / -32

  = - 70 ± 74.43 / -32

By calculating this we get two values:

x = -0.138 and 4.513

Time can never be negative so.

x = 4.513

Therefore the time taken is 4.513 seconds.

To know more about the quadratic function refer to the link given below:

https://brainly.com/question/1214333

#SPJ4

To evaluate forecasting performance of a model or compare relative performance of two
models, examine
a. contingency forecasts
b. conditional forecasts
c. ex ante forecasts
d. ex post forecasts
e. all of these

Answers

Therefore, the answer to this question is option (C) ex ante forecasts

What is  forecasting performance?

To face the challenge of improving operational performance while increasing production efficiency, performance forecasting is a crucial service to help decision-making in the idea, design, and operational phases of an asset.

Here,

As to evaluate forecasting performance of a model or compare relative performance of two models,

can be examined on the basis of ex ante forecasts

The term "ex-ante forecast" refers to a forecast that only takes into account the data (i.e., the value of economic variables) that is available at the time of the actual forecast and not the value of some variables that will be available later on.

Therefore, the answer to this question is option (C) ex ante forecasts

To know more about forecast , visit

https://brainly.com/question/27994086

#SPJ4

Help me pls pls pls plspl splspls

Answers

I will write the answer in the comments please give me a second

a hypothesis test was performed to decide whether houses are more prone to being lost to fire than condominiums are. the researcher looked at 1500 randomly selected houses and 1200 randomly selected condominiums over a ten year period. then, since over 30 of each were in the sample, the researcher can use a z-test for the difference between two proportions.
o False
o True

Answers

It is false, the researcher cannot use a z-test for the difference between two proportions.

A hypothesis test was performed to decide whether houses are more prone to being lost to fire than condominiums are. the researcher looked at 1500 randomly selected houses and 1200 randomly selected condominiums over a ten year period. then, since over 30 of each were in the sample, the researcher cannot use a z-test for the difference between two proportions.

z = (X - mean)/(SD/√n)

Where SD is the standard deviation and n is the number of observations.

Here, the required value is not provided. So, we cannot use the z - test.

Therefore, the researcher cannot use a z-test for the difference between two proportions, so the given statement is false.

To learn more about z test refer here

https://brainly.com/question/14453510

#SPJ4

for each of the three plots, identify the strength of the relationship (e.g. weak, moderate, or strong) in the data and whether fitting a linear model would be reasonable.

Answers

These following answers are correct

We are given residual plots

In given(a) plot there is weak relationship between scatter points

we see curved relationship in this residual plot therefore model is unreasonable .

Note - This model is reasonable when this plot should look approximately normal and scatterplot of residuals should show random

scatter. In these plots we see weak relationship and line as model is unreasonable

2)We are given residual plot.

Given 2nd (c) plot there is strong relationship between scatter points we see positive straigh line relationship in this plot therefore model is resonable.

Also these model look approximately normal land scatterplot of residuals show random scatter.

In these plot we see strong relationship and linear model is reasonable

Therefore, these statements are correct.

To learn more about scatterplot click here:

brainly.com/question/29366075

#SPJ4

In a simple random sample of 144 households in a city in Kentucky, the average number of children in these households was 1.12 children. The standard deviation from this sample was
2.40 children. A 90 percent confidence interval for the mean number of children in all
households in this city is:
Select one:
O a. 1.12 ± 0.2.
O b. 2.40 ± 0.2.
c. It is impossible to tell without a census.
O d. 1.12 ± 2.40.
O e. 1.12 ± 0.328.

Answers

A 90% confidence interval for the mean number of children in all household in this city is 1.12 ± 0.328, using formula for confidence interval.

What are mean and standard deviation ?

The standard deviation is a summary measure of the differences of each  observation from the mean.

Formula to calculate confidence interval using mean and deviation:

mean ± Z*(std deviation/√n)

where n = sample size,

Given that :

mean = 1.12

deviation = 2.40

n = 144

We know Z is fixed for 90% confidence i.e. 1.64

So, the confidence interval = 1.12 ± 1.64*(2.4/√144)

                                              = 1.12 ± 1.64 * .2

                                               = 1.12 ± .328

Learn more about confidence interval using link :

https://brainly.com/question/14825274

#SPJ1

Which statement makes an accurate comparison of the motions for objects C and D?

Answers

Derivatives are used to determine that the following is the best statement:

The acceleration of object C is greater than the acceleration of object D.

When you divide the change in velocity by the change in time, you get the acceleration, which is: a=Δv/Δt

In this issue:

Both object C and object T experience an identical shift in time.The more rapid change in velocity for object C causes it to accelerate more quickly.

hence the right answer is:

The acceleration of object C is greater than the acceleration of object D.

Learn more about on comparison, here:

https://brainly.com/question/9107349

#SPJ4

Answer:

Object C had an acceleration that is greater than the acceleration for D

Step-by-step explanation:

Each person in a random sample of adults was asked how many DVD, he or she wwned. Summary Statistics are given below Variable DVDs N 117 Mean 129.4 Median 50.0 TIMean 76.5 SE Mean StDev 3236 Variable DVDs Minimum 00 Maximum 3000.0 Q1 30.0 03 950 Which of the following statements is true? (A) Seventy-five percent of the adults in the sample own more than 95 DVDs. (B) Fifty percent of the adults in the sample own between 0 and 129.4 DVDs (C) The distribution of the number of DVDs owned appears to be approximately symmetric (D) The interquartile range of the number of DVDs owned is 65. (E) The distribution of the number of DVDs owned contains outliers on both the low side and the high side.

Answers

Hence, the option D "The interquartile range of the number of DVDs owned is 65" is correct.

In the given question, we have to in each person in a random sample of adults was asked how many DVD, he or she owned.

Summary Statistics are given below

Variable     N     Mean     Median     TiMean     St Dev     St Mean

DVDs         117    129.4       50.0          76.5           3236           29.2

Variable     Minimum        Maximum                    Q1            Q3

DVDs              00                3000                         30.0          95.0

We have to which of the following statements is true?

(A) Seventy-five percent of the adults in the sample own more than 95 DVDs.

(B) Fifty percent of the adults in the sample own between 0 and 129.4 DVDs

(C) The distribution of the number of DVDs owned appears to be approximately symmetric

(D) The interquartile range of the number of DVDs owned is 65.

(E) The distribution of the number of DVDs owned contains outliers on both the low side and the high side.

Option (A) is incorrect because 95 is third quartile value and there is only 25% values above third quartile. So, statement is incorrect.

Option (B) is incorrect because 0 is minimum value and 129.4 is mean value. Percent between mean and minimum is not always fixed, so we cannot say that 50% of values are between 0 and 129.4. So, statement is incorrect.

Option (C) is incorrect because median and mean must be equal for symmetric curve, but these are not equal. So, it is not a symmetric curve.

Option (D) is correct because

IQR (Interquartile range)

IQR = Q3 - Q1

IQR = 95 - 30

IQR = 65.

So, option D is correct answer.

Option (E) is incorrect because

lower outlier limit = Q1-1.5*IQR

lower outlier limit = 30 - 1.5*65

lower outlier limit = -67.5

upper outlier limit = Q3 + 1.5*IQR

upper outlier limit = 95 + 1.5*65

upper outlier limit = 192.5

So, there is outliers only on upper side because maximum is 3000>192.5, but there is not value below 0.

To learn more about mean, median and range link is here

brainly.com/question/9026799

#SPJ4

Other Questions
ill give brainly and 10 points if demand for a product is elastic, which one of the following would be true? a. a decrease in price would increase total revenue. b. an increase in price would be total revenue neutral. c. a decrease in price would decrease total revenue. d. an increase in price would increase total revenue. what does atticus mean by ""polite friction""? Which of the German Expressionists was the first to arrive at a totally abstract, non-objective style of representation, often inspired by musical compositions? What is the circumcenter Theorem? What is the equation of a line perpendicular to y= 2z - 6 that passes through (5, 4)Use the formula y = m x + b imagine watching three 4-mo-old females react to a novel environment. sheena and jamie are highly inhibited, while brianne is very low in reactivity (i.e., she is uninhibited). the individual differences in the infants reactions is best captured by the concept of citric acid, the compound responsible for the sour taste of lemons, has the following elemental composition: cc, 37.51%%; hh, 4.20%%; oo, 58.29%%. calculate the empirical formula of citric acid. What is the outcome of protein synthesis?; What is protein synthesis?; What is protein synthesis answer?; What is protein synthesis called? a person with a black belt in karate has a fist that has a mass of 0.70 kg. starting from rest, this fist attains a velocity of 8.0 m/s in 0.15 s. what is the magnitude of the average net force applied to the fist to achieve this level of performance? What is the burden of unintentional injuries? a company that uses the net method of recording purchases and a perpetual inventory system purchased $2,100 of merchandise on july 5 with terms 3/10, n/30. on july 7, it returned $300 worth of merchandise. on july 28, it paid the full amount due. the correct journal entry to record the payment on july 28 is:Multiple Choicea. Debit Merchandise Inventory $1,800; credit Cash $1,800.b. Debit Cash $1,800; credit Accounts Payable $1,800.c. Debit Accounts Payable $1,800; credit Merchandise Inventory $54; credit Cash $1,746.d. Debit Accounts Payable $2,100; credit Cash $2,100.e. Debit Accounts Payable $1,800; credit Cash $1,800. in the event of a wrongful eviction, the tenant is not justified in abandoning the premises without paying rent. true or false? patriot paddleboards sells a paddleboard model that carries a one-year warranty on all included accessories. past experience indicates that 15% of those sold will have defective accessories within a year and that average repair cost is $20 per paddleboard. if 1,000 were sold this year and 50 have already been repaired under warranty, warranty expense for the year would be: Provide, for a set monthly fee, total care with an emphasis on prevention to avoid costly treatment later. chapter 4 review worksheet You want to get the new Iphone. You have two options when purchasing the phone. You can pay a one-time charge of 200$ and then an additional $15 per month . The second option is not paying any money at this time and having a monthly bill of 40$ a month Set up a system of equations to represent this situation. Remeber to define a variable. Find the time in months when both of these plans would pay off the same amout of money for the new phone Why was the New Deal unconstitutional? question 2 a university surveys its student-athletes about their experience in college sports. the survey only includes student-athletes with scholarships. what type of bias is this an example of? 1 point the scale of output that a production facility must operate at to realize all major plant-level scale economies is called the . Which of the following is most likely a poor outcome of Erik Erikson's initiative versus guilt stage of personality development?A. Children becoming resigned to failureB. Children growing up doubting their ability to get along with othersC. Children developing a sense of inferiorityD. Children developing feelings of social discomfort when around others